Limits of functions as x approaches a











up vote
2
down vote

favorite












I need help to find the limits of these two functions :
$$lim_{xto a}frac{x^n-a^n}{x^p-a^p}$$ where $n,p$ are integers.



And :
$$lim_{xto a}{frac{xsin(a)-asin(x)}{x-alog_a(x)}}$$ where $a>0$ and $anotin{{1,e}}$



I can't use L'Hospital rule, any help would be very appreciated !










share|cite|improve this question




















  • 1




    Can you use the definition of derivative?
    – DonAntonio
    Nov 10 at 14:54















up vote
2
down vote

favorite












I need help to find the limits of these two functions :
$$lim_{xto a}frac{x^n-a^n}{x^p-a^p}$$ where $n,p$ are integers.



And :
$$lim_{xto a}{frac{xsin(a)-asin(x)}{x-alog_a(x)}}$$ where $a>0$ and $anotin{{1,e}}$



I can't use L'Hospital rule, any help would be very appreciated !










share|cite|improve this question




















  • 1




    Can you use the definition of derivative?
    – DonAntonio
    Nov 10 at 14:54













up vote
2
down vote

favorite









up vote
2
down vote

favorite











I need help to find the limits of these two functions :
$$lim_{xto a}frac{x^n-a^n}{x^p-a^p}$$ where $n,p$ are integers.



And :
$$lim_{xto a}{frac{xsin(a)-asin(x)}{x-alog_a(x)}}$$ where $a>0$ and $anotin{{1,e}}$



I can't use L'Hospital rule, any help would be very appreciated !










share|cite|improve this question















I need help to find the limits of these two functions :
$$lim_{xto a}frac{x^n-a^n}{x^p-a^p}$$ where $n,p$ are integers.



And :
$$lim_{xto a}{frac{xsin(a)-asin(x)}{x-alog_a(x)}}$$ where $a>0$ and $anotin{{1,e}}$



I can't use L'Hospital rule, any help would be very appreciated !







calculus limits limits-without-lhopital






share|cite|improve this question















share|cite|improve this question













share|cite|improve this question




share|cite|improve this question








edited Nov 10 at 15:10

























asked Nov 10 at 14:53









Cominou

112




112








  • 1




    Can you use the definition of derivative?
    – DonAntonio
    Nov 10 at 14:54














  • 1




    Can you use the definition of derivative?
    – DonAntonio
    Nov 10 at 14:54








1




1




Can you use the definition of derivative?
– DonAntonio
Nov 10 at 14:54




Can you use the definition of derivative?
– DonAntonio
Nov 10 at 14:54










5 Answers
5






active

oldest

votes

















up vote
3
down vote













$$frac{x^n-a^n}{x^p-a^p}=frac{x^n-a^n}{x-a}cdotfrac{x-a}{x^p-a^p}xrightarrow[xto a]{}(x^n)'|_{x=a}cdotfrac1{(x^p)'|_{x=a}}=frac{na^{n-1}}{pa^{p-1}}=frac npa^{n-p}$$






share|cite|improve this answer





















  • The above only uses the definition of derivative at $;x=a;$ for the functions $;x^n,,x^p;$ .
    – DonAntonio
    Nov 10 at 14:58








  • 1




    yes I can, I didn't think about using the definition, thank you !
    – Cominou
    Nov 10 at 15:04










  • @Cominou Excellent. You can repeat exactly the same trick for the other limit...and I think in this second limit it could be $;1-log_ax;$ in he denominator instead...
    – DonAntonio
    Nov 10 at 15:07








  • 1




    i forgot an a in front of the log but i figured it out, it's a-alog(x), sorry :)
    – Cominou
    Nov 10 at 15:15


















up vote
1
down vote













HINT



By $f(x)=x^n$ and $g(x)=x^p$ we have



$$lim_{xto a}frac{x^n-a^n}{x^p-a^p}=lim_{xto a}frac{x^n-a^n}{x-a}frac{x-a}{x^p-a^p}=frac{f'(a)}{g'(a)}$$



and by $f(x)=xsin(a)-asin(x)$ and $g(x)=a-alog_a(x)$ we have



$$lim_{xto a}{frac{xsin(a)-asin(x)}{a-alog_a(x)}}=lim_{xto a}{frac{xsin(a)-asin(x)}{x-a}}{frac{x-a}{a-alog_a(x)}}=frac{f'(a)}{g'(a)}$$






share|cite|improve this answer























  • You are welcome! I've edited for the second limit according to your one. I didn't noticed at first that there was something strange with it, it was indeed trivial in the previous version.
    – gimusi
    Nov 10 at 15:14


















up vote
0
down vote













For the first one you can factor top and bottom and cancel $(x-a)$



For the second one the answer is straight forward because the top goes to zero and the bottom does not.






share|cite|improve this answer





















  • sorry, i forgot the a in front of the log, it's harder like that but i should be able to find the limit thanks to gimusi and donantonio's comments
    – Cominou
    Nov 10 at 15:12


















up vote
0
down vote













The first one simply obtain by L"Hospital rule:
$$lim_{xto a}dfrac{x^n-a^n}{x^p-a^p}=lim_{xto a}dfrac{nx^{n-1}}{px^{p-1}}=dfrac{n}{p}a^{n-p}$$



Edit:
By identity
$$lim_{xto a}dfrac{x^n-a^n}{x^p-a^p}=lim_{xto a}dfrac{(x-a)(x^{n-1}+ax^{n-2}+a^2x^{n-3}+cdots+a^{n-2}x+a^{n-1}}{(x-a)(x^{p-1}+ax^{p-2}+a^2x^{p-3}+cdots+a^{p-2}x+a^{p-1}}=dfrac{n}{p}a^{n-p}$$






share|cite|improve this answer























  • The OP mentioned that he shall not use L'Hospital's Rule.
    – Rebellos
    Nov 10 at 14:57










  • Read carefully at the end of the question...
    – DonAntonio
    Nov 10 at 14:57










  • Yes, I saw tags!
    – Nosrati
    Nov 10 at 14:57










  • @Nosrati "Tags"? The OP writes he cannot use L'Hospital...
    – DonAntonio
    Nov 10 at 14:59


















up vote
0
down vote













For the second limit, we can directly say that it is not indeterminate form so directly put x = a.
We get required limit = $frac{a sin(a) - a sin(a)}{a-1} = 0$



Hope it helps:)






share|cite|improve this answer





















    Your Answer





    StackExchange.ifUsing("editor", function () {
    return StackExchange.using("mathjaxEditing", function () {
    StackExchange.MarkdownEditor.creationCallbacks.add(function (editor, postfix) {
    StackExchange.mathjaxEditing.prepareWmdForMathJax(editor, postfix, [["$", "$"], ["\\(","\\)"]]);
    });
    });
    }, "mathjax-editing");

    StackExchange.ready(function() {
    var channelOptions = {
    tags: "".split(" "),
    id: "69"
    };
    initTagRenderer("".split(" "), "".split(" "), channelOptions);

    StackExchange.using("externalEditor", function() {
    // Have to fire editor after snippets, if snippets enabled
    if (StackExchange.settings.snippets.snippetsEnabled) {
    StackExchange.using("snippets", function() {
    createEditor();
    });
    }
    else {
    createEditor();
    }
    });

    function createEditor() {
    StackExchange.prepareEditor({
    heartbeatType: 'answer',
    convertImagesToLinks: true,
    noModals: true,
    showLowRepImageUploadWarning: true,
    reputationToPostImages: 10,
    bindNavPrevention: true,
    postfix: "",
    imageUploader: {
    brandingHtml: "Powered by u003ca class="icon-imgur-white" href="https://imgur.com/"u003eu003c/au003e",
    contentPolicyHtml: "User contributions licensed under u003ca href="https://creativecommons.org/licenses/by-sa/3.0/"u003ecc by-sa 3.0 with attribution requiredu003c/au003e u003ca href="https://stackoverflow.com/legal/content-policy"u003e(content policy)u003c/au003e",
    allowUrls: true
    },
    noCode: true, onDemand: true,
    discardSelector: ".discard-answer"
    ,immediatelyShowMarkdownHelp:true
    });


    }
    });














     

    draft saved


    draft discarded


















    StackExchange.ready(
    function () {
    StackExchange.openid.initPostLogin('.new-post-login', 'https%3a%2f%2fmath.stackexchange.com%2fquestions%2f2992722%2flimits-of-functions-as-x-approaches-a%23new-answer', 'question_page');
    }
    );

    Post as a guest















    Required, but never shown

























    5 Answers
    5






    active

    oldest

    votes








    5 Answers
    5






    active

    oldest

    votes









    active

    oldest

    votes






    active

    oldest

    votes








    up vote
    3
    down vote













    $$frac{x^n-a^n}{x^p-a^p}=frac{x^n-a^n}{x-a}cdotfrac{x-a}{x^p-a^p}xrightarrow[xto a]{}(x^n)'|_{x=a}cdotfrac1{(x^p)'|_{x=a}}=frac{na^{n-1}}{pa^{p-1}}=frac npa^{n-p}$$






    share|cite|improve this answer





















    • The above only uses the definition of derivative at $;x=a;$ for the functions $;x^n,,x^p;$ .
      – DonAntonio
      Nov 10 at 14:58








    • 1




      yes I can, I didn't think about using the definition, thank you !
      – Cominou
      Nov 10 at 15:04










    • @Cominou Excellent. You can repeat exactly the same trick for the other limit...and I think in this second limit it could be $;1-log_ax;$ in he denominator instead...
      – DonAntonio
      Nov 10 at 15:07








    • 1




      i forgot an a in front of the log but i figured it out, it's a-alog(x), sorry :)
      – Cominou
      Nov 10 at 15:15















    up vote
    3
    down vote













    $$frac{x^n-a^n}{x^p-a^p}=frac{x^n-a^n}{x-a}cdotfrac{x-a}{x^p-a^p}xrightarrow[xto a]{}(x^n)'|_{x=a}cdotfrac1{(x^p)'|_{x=a}}=frac{na^{n-1}}{pa^{p-1}}=frac npa^{n-p}$$






    share|cite|improve this answer





















    • The above only uses the definition of derivative at $;x=a;$ for the functions $;x^n,,x^p;$ .
      – DonAntonio
      Nov 10 at 14:58








    • 1




      yes I can, I didn't think about using the definition, thank you !
      – Cominou
      Nov 10 at 15:04










    • @Cominou Excellent. You can repeat exactly the same trick for the other limit...and I think in this second limit it could be $;1-log_ax;$ in he denominator instead...
      – DonAntonio
      Nov 10 at 15:07








    • 1




      i forgot an a in front of the log but i figured it out, it's a-alog(x), sorry :)
      – Cominou
      Nov 10 at 15:15













    up vote
    3
    down vote










    up vote
    3
    down vote









    $$frac{x^n-a^n}{x^p-a^p}=frac{x^n-a^n}{x-a}cdotfrac{x-a}{x^p-a^p}xrightarrow[xto a]{}(x^n)'|_{x=a}cdotfrac1{(x^p)'|_{x=a}}=frac{na^{n-1}}{pa^{p-1}}=frac npa^{n-p}$$






    share|cite|improve this answer












    $$frac{x^n-a^n}{x^p-a^p}=frac{x^n-a^n}{x-a}cdotfrac{x-a}{x^p-a^p}xrightarrow[xto a]{}(x^n)'|_{x=a}cdotfrac1{(x^p)'|_{x=a}}=frac{na^{n-1}}{pa^{p-1}}=frac npa^{n-p}$$







    share|cite|improve this answer












    share|cite|improve this answer



    share|cite|improve this answer










    answered Nov 10 at 14:57









    DonAntonio

    175k1491224




    175k1491224












    • The above only uses the definition of derivative at $;x=a;$ for the functions $;x^n,,x^p;$ .
      – DonAntonio
      Nov 10 at 14:58








    • 1




      yes I can, I didn't think about using the definition, thank you !
      – Cominou
      Nov 10 at 15:04










    • @Cominou Excellent. You can repeat exactly the same trick for the other limit...and I think in this second limit it could be $;1-log_ax;$ in he denominator instead...
      – DonAntonio
      Nov 10 at 15:07








    • 1




      i forgot an a in front of the log but i figured it out, it's a-alog(x), sorry :)
      – Cominou
      Nov 10 at 15:15


















    • The above only uses the definition of derivative at $;x=a;$ for the functions $;x^n,,x^p;$ .
      – DonAntonio
      Nov 10 at 14:58








    • 1




      yes I can, I didn't think about using the definition, thank you !
      – Cominou
      Nov 10 at 15:04










    • @Cominou Excellent. You can repeat exactly the same trick for the other limit...and I think in this second limit it could be $;1-log_ax;$ in he denominator instead...
      – DonAntonio
      Nov 10 at 15:07








    • 1




      i forgot an a in front of the log but i figured it out, it's a-alog(x), sorry :)
      – Cominou
      Nov 10 at 15:15
















    The above only uses the definition of derivative at $;x=a;$ for the functions $;x^n,,x^p;$ .
    – DonAntonio
    Nov 10 at 14:58






    The above only uses the definition of derivative at $;x=a;$ for the functions $;x^n,,x^p;$ .
    – DonAntonio
    Nov 10 at 14:58






    1




    1




    yes I can, I didn't think about using the definition, thank you !
    – Cominou
    Nov 10 at 15:04




    yes I can, I didn't think about using the definition, thank you !
    – Cominou
    Nov 10 at 15:04












    @Cominou Excellent. You can repeat exactly the same trick for the other limit...and I think in this second limit it could be $;1-log_ax;$ in he denominator instead...
    – DonAntonio
    Nov 10 at 15:07






    @Cominou Excellent. You can repeat exactly the same trick for the other limit...and I think in this second limit it could be $;1-log_ax;$ in he denominator instead...
    – DonAntonio
    Nov 10 at 15:07






    1




    1




    i forgot an a in front of the log but i figured it out, it's a-alog(x), sorry :)
    – Cominou
    Nov 10 at 15:15




    i forgot an a in front of the log but i figured it out, it's a-alog(x), sorry :)
    – Cominou
    Nov 10 at 15:15










    up vote
    1
    down vote













    HINT



    By $f(x)=x^n$ and $g(x)=x^p$ we have



    $$lim_{xto a}frac{x^n-a^n}{x^p-a^p}=lim_{xto a}frac{x^n-a^n}{x-a}frac{x-a}{x^p-a^p}=frac{f'(a)}{g'(a)}$$



    and by $f(x)=xsin(a)-asin(x)$ and $g(x)=a-alog_a(x)$ we have



    $$lim_{xto a}{frac{xsin(a)-asin(x)}{a-alog_a(x)}}=lim_{xto a}{frac{xsin(a)-asin(x)}{x-a}}{frac{x-a}{a-alog_a(x)}}=frac{f'(a)}{g'(a)}$$






    share|cite|improve this answer























    • You are welcome! I've edited for the second limit according to your one. I didn't noticed at first that there was something strange with it, it was indeed trivial in the previous version.
      – gimusi
      Nov 10 at 15:14















    up vote
    1
    down vote













    HINT



    By $f(x)=x^n$ and $g(x)=x^p$ we have



    $$lim_{xto a}frac{x^n-a^n}{x^p-a^p}=lim_{xto a}frac{x^n-a^n}{x-a}frac{x-a}{x^p-a^p}=frac{f'(a)}{g'(a)}$$



    and by $f(x)=xsin(a)-asin(x)$ and $g(x)=a-alog_a(x)$ we have



    $$lim_{xto a}{frac{xsin(a)-asin(x)}{a-alog_a(x)}}=lim_{xto a}{frac{xsin(a)-asin(x)}{x-a}}{frac{x-a}{a-alog_a(x)}}=frac{f'(a)}{g'(a)}$$






    share|cite|improve this answer























    • You are welcome! I've edited for the second limit according to your one. I didn't noticed at first that there was something strange with it, it was indeed trivial in the previous version.
      – gimusi
      Nov 10 at 15:14













    up vote
    1
    down vote










    up vote
    1
    down vote









    HINT



    By $f(x)=x^n$ and $g(x)=x^p$ we have



    $$lim_{xto a}frac{x^n-a^n}{x^p-a^p}=lim_{xto a}frac{x^n-a^n}{x-a}frac{x-a}{x^p-a^p}=frac{f'(a)}{g'(a)}$$



    and by $f(x)=xsin(a)-asin(x)$ and $g(x)=a-alog_a(x)$ we have



    $$lim_{xto a}{frac{xsin(a)-asin(x)}{a-alog_a(x)}}=lim_{xto a}{frac{xsin(a)-asin(x)}{x-a}}{frac{x-a}{a-alog_a(x)}}=frac{f'(a)}{g'(a)}$$






    share|cite|improve this answer














    HINT



    By $f(x)=x^n$ and $g(x)=x^p$ we have



    $$lim_{xto a}frac{x^n-a^n}{x^p-a^p}=lim_{xto a}frac{x^n-a^n}{x-a}frac{x-a}{x^p-a^p}=frac{f'(a)}{g'(a)}$$



    and by $f(x)=xsin(a)-asin(x)$ and $g(x)=a-alog_a(x)$ we have



    $$lim_{xto a}{frac{xsin(a)-asin(x)}{a-alog_a(x)}}=lim_{xto a}{frac{xsin(a)-asin(x)}{x-a}}{frac{x-a}{a-alog_a(x)}}=frac{f'(a)}{g'(a)}$$







    share|cite|improve this answer














    share|cite|improve this answer



    share|cite|improve this answer








    edited Nov 10 at 15:10

























    answered Nov 10 at 14:57









    gimusi

    85.8k74294




    85.8k74294












    • You are welcome! I've edited for the second limit according to your one. I didn't noticed at first that there was something strange with it, it was indeed trivial in the previous version.
      – gimusi
      Nov 10 at 15:14


















    • You are welcome! I've edited for the second limit according to your one. I didn't noticed at first that there was something strange with it, it was indeed trivial in the previous version.
      – gimusi
      Nov 10 at 15:14
















    You are welcome! I've edited for the second limit according to your one. I didn't noticed at first that there was something strange with it, it was indeed trivial in the previous version.
    – gimusi
    Nov 10 at 15:14




    You are welcome! I've edited for the second limit according to your one. I didn't noticed at first that there was something strange with it, it was indeed trivial in the previous version.
    – gimusi
    Nov 10 at 15:14










    up vote
    0
    down vote













    For the first one you can factor top and bottom and cancel $(x-a)$



    For the second one the answer is straight forward because the top goes to zero and the bottom does not.






    share|cite|improve this answer





















    • sorry, i forgot the a in front of the log, it's harder like that but i should be able to find the limit thanks to gimusi and donantonio's comments
      – Cominou
      Nov 10 at 15:12















    up vote
    0
    down vote













    For the first one you can factor top and bottom and cancel $(x-a)$



    For the second one the answer is straight forward because the top goes to zero and the bottom does not.






    share|cite|improve this answer





















    • sorry, i forgot the a in front of the log, it's harder like that but i should be able to find the limit thanks to gimusi and donantonio's comments
      – Cominou
      Nov 10 at 15:12













    up vote
    0
    down vote










    up vote
    0
    down vote









    For the first one you can factor top and bottom and cancel $(x-a)$



    For the second one the answer is straight forward because the top goes to zero and the bottom does not.






    share|cite|improve this answer












    For the first one you can factor top and bottom and cancel $(x-a)$



    For the second one the answer is straight forward because the top goes to zero and the bottom does not.







    share|cite|improve this answer












    share|cite|improve this answer



    share|cite|improve this answer










    answered Nov 10 at 15:01









    Mohammad Riazi-Kermani

    40.2k41958




    40.2k41958












    • sorry, i forgot the a in front of the log, it's harder like that but i should be able to find the limit thanks to gimusi and donantonio's comments
      – Cominou
      Nov 10 at 15:12


















    • sorry, i forgot the a in front of the log, it's harder like that but i should be able to find the limit thanks to gimusi and donantonio's comments
      – Cominou
      Nov 10 at 15:12
















    sorry, i forgot the a in front of the log, it's harder like that but i should be able to find the limit thanks to gimusi and donantonio's comments
    – Cominou
    Nov 10 at 15:12




    sorry, i forgot the a in front of the log, it's harder like that but i should be able to find the limit thanks to gimusi and donantonio's comments
    – Cominou
    Nov 10 at 15:12










    up vote
    0
    down vote













    The first one simply obtain by L"Hospital rule:
    $$lim_{xto a}dfrac{x^n-a^n}{x^p-a^p}=lim_{xto a}dfrac{nx^{n-1}}{px^{p-1}}=dfrac{n}{p}a^{n-p}$$



    Edit:
    By identity
    $$lim_{xto a}dfrac{x^n-a^n}{x^p-a^p}=lim_{xto a}dfrac{(x-a)(x^{n-1}+ax^{n-2}+a^2x^{n-3}+cdots+a^{n-2}x+a^{n-1}}{(x-a)(x^{p-1}+ax^{p-2}+a^2x^{p-3}+cdots+a^{p-2}x+a^{p-1}}=dfrac{n}{p}a^{n-p}$$






    share|cite|improve this answer























    • The OP mentioned that he shall not use L'Hospital's Rule.
      – Rebellos
      Nov 10 at 14:57










    • Read carefully at the end of the question...
      – DonAntonio
      Nov 10 at 14:57










    • Yes, I saw tags!
      – Nosrati
      Nov 10 at 14:57










    • @Nosrati "Tags"? The OP writes he cannot use L'Hospital...
      – DonAntonio
      Nov 10 at 14:59















    up vote
    0
    down vote













    The first one simply obtain by L"Hospital rule:
    $$lim_{xto a}dfrac{x^n-a^n}{x^p-a^p}=lim_{xto a}dfrac{nx^{n-1}}{px^{p-1}}=dfrac{n}{p}a^{n-p}$$



    Edit:
    By identity
    $$lim_{xto a}dfrac{x^n-a^n}{x^p-a^p}=lim_{xto a}dfrac{(x-a)(x^{n-1}+ax^{n-2}+a^2x^{n-3}+cdots+a^{n-2}x+a^{n-1}}{(x-a)(x^{p-1}+ax^{p-2}+a^2x^{p-3}+cdots+a^{p-2}x+a^{p-1}}=dfrac{n}{p}a^{n-p}$$






    share|cite|improve this answer























    • The OP mentioned that he shall not use L'Hospital's Rule.
      – Rebellos
      Nov 10 at 14:57










    • Read carefully at the end of the question...
      – DonAntonio
      Nov 10 at 14:57










    • Yes, I saw tags!
      – Nosrati
      Nov 10 at 14:57










    • @Nosrati "Tags"? The OP writes he cannot use L'Hospital...
      – DonAntonio
      Nov 10 at 14:59













    up vote
    0
    down vote










    up vote
    0
    down vote









    The first one simply obtain by L"Hospital rule:
    $$lim_{xto a}dfrac{x^n-a^n}{x^p-a^p}=lim_{xto a}dfrac{nx^{n-1}}{px^{p-1}}=dfrac{n}{p}a^{n-p}$$



    Edit:
    By identity
    $$lim_{xto a}dfrac{x^n-a^n}{x^p-a^p}=lim_{xto a}dfrac{(x-a)(x^{n-1}+ax^{n-2}+a^2x^{n-3}+cdots+a^{n-2}x+a^{n-1}}{(x-a)(x^{p-1}+ax^{p-2}+a^2x^{p-3}+cdots+a^{p-2}x+a^{p-1}}=dfrac{n}{p}a^{n-p}$$






    share|cite|improve this answer














    The first one simply obtain by L"Hospital rule:
    $$lim_{xto a}dfrac{x^n-a^n}{x^p-a^p}=lim_{xto a}dfrac{nx^{n-1}}{px^{p-1}}=dfrac{n}{p}a^{n-p}$$



    Edit:
    By identity
    $$lim_{xto a}dfrac{x^n-a^n}{x^p-a^p}=lim_{xto a}dfrac{(x-a)(x^{n-1}+ax^{n-2}+a^2x^{n-3}+cdots+a^{n-2}x+a^{n-1}}{(x-a)(x^{p-1}+ax^{p-2}+a^2x^{p-3}+cdots+a^{p-2}x+a^{p-1}}=dfrac{n}{p}a^{n-p}$$







    share|cite|improve this answer














    share|cite|improve this answer



    share|cite|improve this answer








    edited Nov 10 at 15:02

























    answered Nov 10 at 14:56









    Nosrati

    25.8k62252




    25.8k62252












    • The OP mentioned that he shall not use L'Hospital's Rule.
      – Rebellos
      Nov 10 at 14:57










    • Read carefully at the end of the question...
      – DonAntonio
      Nov 10 at 14:57










    • Yes, I saw tags!
      – Nosrati
      Nov 10 at 14:57










    • @Nosrati "Tags"? The OP writes he cannot use L'Hospital...
      – DonAntonio
      Nov 10 at 14:59


















    • The OP mentioned that he shall not use L'Hospital's Rule.
      – Rebellos
      Nov 10 at 14:57










    • Read carefully at the end of the question...
      – DonAntonio
      Nov 10 at 14:57










    • Yes, I saw tags!
      – Nosrati
      Nov 10 at 14:57










    • @Nosrati "Tags"? The OP writes he cannot use L'Hospital...
      – DonAntonio
      Nov 10 at 14:59
















    The OP mentioned that he shall not use L'Hospital's Rule.
    – Rebellos
    Nov 10 at 14:57




    The OP mentioned that he shall not use L'Hospital's Rule.
    – Rebellos
    Nov 10 at 14:57












    Read carefully at the end of the question...
    – DonAntonio
    Nov 10 at 14:57




    Read carefully at the end of the question...
    – DonAntonio
    Nov 10 at 14:57












    Yes, I saw tags!
    – Nosrati
    Nov 10 at 14:57




    Yes, I saw tags!
    – Nosrati
    Nov 10 at 14:57












    @Nosrati "Tags"? The OP writes he cannot use L'Hospital...
    – DonAntonio
    Nov 10 at 14:59




    @Nosrati "Tags"? The OP writes he cannot use L'Hospital...
    – DonAntonio
    Nov 10 at 14:59










    up vote
    0
    down vote













    For the second limit, we can directly say that it is not indeterminate form so directly put x = a.
    We get required limit = $frac{a sin(a) - a sin(a)}{a-1} = 0$



    Hope it helps:)






    share|cite|improve this answer

























      up vote
      0
      down vote













      For the second limit, we can directly say that it is not indeterminate form so directly put x = a.
      We get required limit = $frac{a sin(a) - a sin(a)}{a-1} = 0$



      Hope it helps:)






      share|cite|improve this answer























        up vote
        0
        down vote










        up vote
        0
        down vote









        For the second limit, we can directly say that it is not indeterminate form so directly put x = a.
        We get required limit = $frac{a sin(a) - a sin(a)}{a-1} = 0$



        Hope it helps:)






        share|cite|improve this answer












        For the second limit, we can directly say that it is not indeterminate form so directly put x = a.
        We get required limit = $frac{a sin(a) - a sin(a)}{a-1} = 0$



        Hope it helps:)







        share|cite|improve this answer












        share|cite|improve this answer



        share|cite|improve this answer










        answered Nov 10 at 15:06









        Crazy for maths

        4948




        4948






























             

            draft saved


            draft discarded



















































             


            draft saved


            draft discarded














            StackExchange.ready(
            function () {
            StackExchange.openid.initPostLogin('.new-post-login', 'https%3a%2f%2fmath.stackexchange.com%2fquestions%2f2992722%2flimits-of-functions-as-x-approaches-a%23new-answer', 'question_page');
            }
            );

            Post as a guest















            Required, but never shown





















































            Required, but never shown














            Required, but never shown












            Required, but never shown







            Required, but never shown

































            Required, but never shown














            Required, but never shown












            Required, but never shown







            Required, but never shown







            Popular posts from this blog

            Florida Star v. B. J. F.

            Error while running script in elastic search , gateway timeout

            Adding quotations to stringified JSON object values